Getting the Horizontal Asymptotes

Click For Summary
The discussion focuses on understanding horizontal asymptotes, particularly for the function f(x) = 1/(1+e^(3x)). Participants note the absence of vertical asymptotes and emphasize the importance of graphing the function to analyze its behavior. Key questions are raised about the behavior of e^(3x) as x approaches large positive and negative values. The clarification of the function's denominator is also highlighted as essential for accurate analysis. Overall, the conversation aims to deepen understanding of horizontal asymptotes in the context of exponential functions.
MatthewR

Homework Statement


upload_2017-12-12_21-42-30.png


Homework Equations

The Attempt at a Solution


I understand there is no vertical asymptotes and can usually get the horizontal ,but can't understand with the exponential.
 

Attachments

  • upload_2017-12-12_21-42-30.png
    upload_2017-12-12_21-42-30.png
    1.5 KB · Views: 828
Physics news on Phys.org
MatthewR said:

Homework Statement


View attachment 216624

Homework Equations

The Attempt at a Solution


I understand there is no vertical asymptotes and can usually get the horizontal ,but can't understand with the exponential.

Your image is fuzzy and unreadable. Take the trouble to actually type out the problem---that is actually the PF preferred standard!
 
  • Like
Likes member 587159
Ray Vickson said:
Your image is fuzzy and unreadable. Take the trouble to actually type out the problem---that is actually the PF preferred standard!
Noted :) , here goes:

f(x)=1/1+e3x
 
Try graphing it, and then go from there.
 
MatthewR said:
Noted :) , here goes:

f(x)=1/1+e3x
You should enclose that denominator in parentheses.
f(x)=1/(1+e3x)​

What happens to e3x as x gets very large ?

What happens to e3x as x gets very negative ?
 
Question: A clock's minute hand has length 4 and its hour hand has length 3. What is the distance between the tips at the moment when it is increasing most rapidly?(Putnam Exam Question) Answer: Making assumption that both the hands moves at constant angular velocities, the answer is ## \sqrt{7} .## But don't you think this assumption is somewhat doubtful and wrong?

Similar threads

  • · Replies 6 ·
Replies
6
Views
1K
  • · Replies 5 ·
Replies
5
Views
2K
  • · Replies 5 ·
Replies
5
Views
2K
  • · Replies 10 ·
Replies
10
Views
2K
  • · Replies 12 ·
Replies
12
Views
2K
Replies
4
Views
2K
  • · Replies 2 ·
Replies
2
Views
3K
  • · Replies 4 ·
Replies
4
Views
2K
Replies
11
Views
2K
  • · Replies 1 ·
Replies
1
Views
5K